2020-2021/TST/DM/2102_DM2/TST3/09_2102_DM2.tex

204 lines
7.5 KiB
TeX

\documentclass[a5paper,10pt]{article}
\usepackage{myXsim}
\usepackage{tasks}
% Title Page
\title{DM2 \hfill DEBRAS Noémie}
\tribe{TST}
\date{\hfillÀ render pour le Mercredi 24 février}
\xsimsetup{
solution/print = false
}
\begin{document}
\maketitle
\begin{exercise}[subtitle={Loi binomiale}]
Trois personnes s'apprêtent à passer le portique de sécurité. On suppose que pour chaque personne la probabilité que le portique sonne est égale à $0.1$.
Soit $X$ la variable aléatoire donnant le nombre de personnes faisant sonner le portique, parmi les 3 personnes de ce groupe.
\begin{enumerate}
\item Tracer l'arbre représentant le situation.
\item Justifier que $X$ suit une loi binomiale dont on précisera les paramètres.
\item Quelle est la probabilité qu'une seule personne fasse sonner le portique?
\item Calculer puis interpréter les probabilités suivantes
\[
P(X = 0) \qquad \qquad P(X \geq 2)
\]
\item Calculer l'espérance de $X$ et interpréter le résultat.
\end{enumerate}
\end{exercise}
\begin{solution}
\begin{enumerate}
\item
\begin{tikzpicture}[sloped]
\node {.}
child {node {$0$}
child {node {$0$}
child {node {$0$}
edge from parent
node[above] {0.9}
}
child {node {$1$}
edge from parent
node[above] {0.1}
}
edge from parent
node[above] {0.9}
}
child[missing] {}
child {node {$1$}
child {node {$0$}
edge from parent
node[above] {0.9}
}
child {node {$1$}
edge from parent
node[above] {0.1}
}
edge from parent
node[above] {0.9}
}
edge from parent
node[above] {0.9}
}
child[missing] {}
child[missing] {}
child[missing] {}
child { node {$1$}
child {node {$0$}
child {node {$0$}
edge from parent
node[above] {0.9}
}
child {node {$1$}
edge from parent
node[above] {0.1}
}
edge from parent
node[above] {0.9}
}
child[missing] {}
child {node {$1$}
child {node {$0$}
edge from parent
node[above] {0.9}
}
child {node {$1$}
edge from parent
node[above] {0.1}
}
edge from parent
node[above] {0.9}
}
edge from parent
node[above] {0.1}
} ;
\end{tikzpicture}
\item Chaque personne a 2 possibilités (1: fait sonner ou 2: ne fait pas sonner) et l'on fait passer 3 personnes ce qui correspond à une répétition identique et aléatoire. On peut donc modéliser la situation par une loi binomiale.
\[
X \sim \mathcal{B}(3; 0.76)
\]
\item Probabilité qu'une seule personne fasse sonner le portique. On voit qu'il y a 3 branches qui correspondent à cette situation dont
\[
P(X = 1) = 3 \times 0.1^1 \times 0.9^2 \approx 0.243
\]
\item
\[
P(X = 0) = 0.9^3 \approx 0.729
\]
\[
P(X \geq 2) = P(X = 2) + P(X = 3) = 3 \times 0.1^2 \times 0.9^1 + 0.1^3 \approx 0.028
\]
\item Il faut d'abord tracer le tableau résumant la loi de probabilité:
\begin{center}
\begin{tabular}{|c|*{4}{c|}}
\hline
Valeur & 0 & 1 & 2 & 3 \\
\hline
Probabilité & $0.729$ & $0.243$ & $0.027$ &$0.001$ \\
\hline
\end{tabular}
\end{center}
On peut alors calculer l'espérance
\[
E[X] = 0 \times 0.729 + 1 \times 0.243 + 2 \times 0.027 + 3 \times 0.001 = 0.3
\]
On peut donc estimer qu'il y aura en moyenne $0.3$ personnes qui feront sonner le portique sur les 3 personnes.
\end{enumerate}
\end{solution}
\begin{exercise}[subtitle={Équation puissance}]
Résoudre les équations et inéquations suivantes
\begin{multicols}{2}
\begin{enumerate}
\item $10^x = 25$
\item $12^x = 21$
\item $0.77^x \leq 22$
\item $6 \times 0.4^x = 40$
\end{enumerate}
\end{multicols}
\end{exercise}
\begin{solution}
Les solutions ci-dessous ne sont pas justifiée car l'ordinateur ne sait pas faire. Par contre, vous vous devez savoir justifier vos réponses!
\begin{enumerate}
\item $x = \log(25)$
\item $x = \frac{\log(21)}{\log(12)}$
\item Il faut faire attention quand on divise par un log car ce dernier peut être négatif ce qui est le cas ici. Il faut donc pense à changer le sens de l'inégalité.
$x \geq \frac{\log(22)}{\log(0.77)}$
\item Il faut penser à faire la division à par $6$ avant d'utiliser le log car sinon, on ne peut pas utiliser la formule $\log(a^n) = n\times \log(a)$.
$x = \frac{\log(6.67)}{\log(0.4)}$
\end{enumerate}
\end{solution}
\begin{exercise}[subtitle={Étude de fonctions}]
Soit $f(x) = 4x^3 - 132x^2 - 5460x + 39$ une fonction définie sur $\R$.
\begin{enumerate}
\item Calculer $f'(x)$ la dérivée de $f(x)$.
\item Calculer $f'(35)$ et $f'(-13)$.
\item En déduire une forme factorisée de $f'(x)$.
\item Étudier le signe de $f'(x)$ et en déduire les variations de $f(x)$.
\item Est-ce que la fonction $f(x)$ admet un maximum ou un minimum? Si oui, calculer sa valeur.
\end{enumerate}
\end{exercise}
\begin{solution}
\begin{enumerate}
\item Dérivée de $f(x)$: $f'(x) = 12x^2 - 264x - 5460$
\item
\begin{align*}
f'(35) &= 12 \times 35^{2} - 264 \times 35 - 5460\\&= 12 \times 1225 - 9240 - 5460\\&= 14700 - 14700\\&= 0
\end{align*}
\begin{align*}
f'(-13) &= 12 \times - 13^{2} - 264(- 13) - 5460\\&= 12 \times 169 + 3432 - 5460\\&= 2028 - 2028\\&= 0
\end{align*}
Donc $x = 35$ et $x=-13$ sont des racines de $f'(x) = 12x^2 - 264x - 5460$.
\item On en déduit la forme factorisée suivante
\[
f'(x) = 12 (x - 35)(x--13)
\]
\item Pas de correction disponible
\item À causes des branches extérieurs, la fonction $f(x)$ n'a pas de maximum ou de minimum.
\end{enumerate}
\end{solution}
%\printsolutionstype{exercise}
\end{document}
%%% Local Variables:
%%% mode: latex
%%% TeX-master: "master"
%%% End: